Which answer choice gives the correct surface area for a triangular prism
with bases that are 4 cm² and sides that are 4 cm²?

Answers

Answer 1

Answer:

The answer is 20

Step-by-step explanation:

Their are two bases and three sides in a triangular prism so the total surface area is 20


Related Questions

60 POINTS PLEASEEEEEEEEEEEE

Answers

Answer:

A.) Two

Step-by-step explanation:

You can determine the amount of solutions the functions have by counting the number of intersections they have. Since the functions cross each other at two different points, there are two solutions.

The answer is A, twosolutions

Identify each of the following as rational or irrational.

Answers

Answer:A rational number can be written as a fraction of two integers. √23 is irrational because we do not know what the square root of 23 is in terms of integers in a fraction. 104.42 is rational because it can be expressed as 10442/100 . We can express it as this because.

Step-by-step explanation:

Solve for t 16−2t=t+9+4t

Answers

Answer:

Step-by-step explanation:

16-2t=t+9+4t  

7=7t  
t=1

You enter a room. 2 dogs, 4 horses, one giraffe, and a duck are laying on a bed. 3 chicken are flying over a chair. How many legs are on the ground?.

Answers

Answer: the answer is 28

Step-by-step explanation:

I can walk at 6km/hr and run at 15km/hr. On a journey I spend as long walking as I do running. If I had walked twice as long on the journey it would have taken me six minutes longer find how far I ran​

Answers

He ran for 12 km in his journey.

What is an Equation ?

An equation is formed when two algebraic expressions are equated by an equal sign.

It is given that

Speed of Walking =6km/hr  

Speed of Running = 15km/hr

Let he walks and run x hours ( as given)

he covered distance = 21 x km

The total time taken in the journey is 2x

for the same distance

It is also given that

if for the total journey

if he walks walked twice as long on the journey

He walked 6x in the journey so new distance that he walked is  12x  , then the total journey time = 6 minutes longer

Converting speed from km / hr to km/ min

1 km / hr = 60 km/ min

and forming an equation with the given data

12x * 60 /6 + 9x *60/15 = 2x * 60 + 6

120x + 36x = 120x +6

150x = 120

x = 120/150

x = 0.8 hours.

Therefore he spent 0.8 hours walking and 0.8 hours running and He ran 15 * 0.8 = 12 km

To know more about Equation

https://brainly.com/question/10413253

#SPJ1

Which is the graph of f(x) = 4( 1 /2)x

Answers

Answer:

I don't see any option to choose from.

Step-by-step explanation:

See attached graph.

f(x) = 4( 1 /2)x can be rewritten as y = 2x.  This is a straight line with slope 2 and a y-intercept of 0.

PLEASE HELP!!!!
Use the following set of data:

78, 78, 83, 85, 89, 91, 95, 98
What is the median for the set of data?
(A) 85
(B) 89
(C) 87
(D) 98

Answers

Answer:

87

Step-by-step explanation:

The median of a data set is the value that is exactly in the middle of all other values.

To find the median of a data set, we typically list the values in order of greatest to least, and we look for the number in the middle.

Here, the list-maker has already provided the list in this order!

78, 78, 83, 85, 89, 91, 95, 98

we can see two numbers exactly in the middle here--when this happens, we take the mean/average of the two:

85 + 89 = 174

174 / 2 = 87

So, the median of this data set is 87

hope this helps! have a lovely day :)

87, add all numbers together

To get 697, then divide by amount on numbers (8)

To get 87.125 then round to the nearest whole number

To get 87

Miss Goh bought 6 blouses, 2 skirts and 2 dresses for $560. A blouse and a skirt cost $107. A blouse and a dress cost $134. What was the cost of a dress?

Answers

Answer:

$95 FOR A DRESS

Step-by-step explanation:

Let's rearrange the given:

  She bought a blouse and skirt for two, a blouse and a dress for two, and a blouse for two.

     A blouse and a skirt cost $107

     A blouse and a dress cost $134

     blouse cost x dollars

The equation:

     2($107) + 2($134) + 2x = $560

SOLUTION:

   $214 + $268 + 2x = $560

   2x = $560 - $214 - $268

   2x = $78

   x = $39 for a single blouse

FIND THE COST OF DRESS:

   A blouse and a dress cost $134 - $39 for a single blouse

      $134 - $39 = $95

$95 IS THE COST OF DRESS

   

PLEASE HELP URGENT

B Pick the inequality that matches the graph shown. Show all of your work to explain the
choice you selected. (See Ex. 1)

Answers

Answer:

  B.  y > 3x -7

Step-by-step explanation:

We can choose the correct inequality based on the y-intercept and the shading.

Y-intercept

The dashed line crosses the y-axis at approximately y=-7. This is the value of y when x=0. This observation eliminates choices D, E, F.

Shading

The shading on the graph is seen to be above and to the left of the line. This means the variables in relation to the inequality symbol must be ...

  y > ( ) . . . . eliminates choice A

and

  x < ( )  or  ( ) > x . . . . eliminates choice C, confirms choice B

The inequality that matches the graph is y > 3x -7.

__

Additional comment

For looking at shading, we are interested in the relation of the inequality symbol to a variable term with a positive coefficient. If the coefficient is negative, you can do either of ...

add the opposite of that variable term to both sides of the equationreverse the inequality symbol

That is, if you have -3x < ( ), when considering shading, you can consider the relation x > ( ) with the inequality reversed, or you can consider ( ) < x, which has x on the other side of the inequality. Both of these tell you shading is right of the line, where x-values are greater than those on the line.

Y=
Help me please thank u

Answers

Answer:

Y≈4.6

Step-by-step explanation:

Because the hypotenuse is 9rad2 and the angle opposite of it is a 90. You can deduce it is a 90-45-45. From this you can tell the legs are 9. You then do 9 tan 26 to get about 4.4, and after this you subtract it from 9.

Trini rode her bike 12 miles on Friday. She rode 14 miles on Saturday and 15 miles on Sunday. How many miles did she ride in all?​

Answers

Answer: 41

Step-by-step explanation:

12+14+15= 41

A fence is to be built to enclose a rectangular area of 200 square feet. The fence along three sides is to be made of material that costs 3 dollars per foot, and the material for the fourth side costs 12 dollars per foot. Find the dimensions of the enclosure that is most economical to construct.

Answers

y = 220/ ((10/3) * √22)

It is given that area is 200 square feet. Cost is 3 dollar per foot. Fourth side costs 12 dollars per foot.

This f(x,y) needs to represent the cost of the fence so we can look at each side's price. We can choose the fourth side to be along the x axis and be represented by 13x. The other sides therefore must be represented by 5y, 5y, and 5x.

So, our cost, f(x,y) = 13x + 5x + 5y + 5y = 18x + 10y

Our constraint is xy = 220 as defined by the area of a rectangle.

We can then take our constraint to be put in terms of exclusively x, giving us y = 220/x

Plugging this into our cost, the thing we are minimizing, we get f(x) = 18x + 2200/x

In order to find the minimum we use the first derivative test, taking f'(x) and finding the critical points.

f'(x) = 18 - 2200/x2

Setting this equation to be equal to 0 we find that x = ±√2200/18 . But the negative answer doesn't make sense because distance cannot be negative so we throw it out.

x = (10/3) * √22

We must verify that this is a minimum by confirming the following:

If x < (10/3) * √22, f'(x) < 0. So, f(x) is decreasing when x < (10/3) * √22.

If x > (10/3) * √22, f'(x) > 0. So, f(x) is increasing when x > (10/3) * √22.

Thus, we have guaranteed that (10/3) * √22 is the x dimension. Now we plug in this value in our original equation to find y and that is our y dimension. So, y = 220/ ((10/3) * √22)

Learn more about word problems here: brainly.com/question/21405634

#SPJ4

-2x
10. What is the least common denominator of 21+2+³=1?
+3
O(x - 1)(x+1)
Ox(x - 1)2(x + 1)
O(x-1)2(x + 1)
Ox(x - 1)(x + 1)

Answers

Answer:

O(x-1)2(x + 1)

Step-by-step explanation:

35. For the following exercises, given each set of information, find a linear equation satisfying the conditions, if possible.
35. Passes through (-2, 8) and (4, 6)

Answers

Answer:

The linear equation for the line which passes through the points given as (-2,8) and $(4,6), is written in the point-slope form as [tex]$y=-\frac{1}{3} x-\frac{26}{3}$[/tex].

Step-by-step explanation:

A condition is given that a line passes through the points whose coordinates are (-2,8) and (4,6).

It is asked to find the linear equation which satisfies the given condition.

Step 1 of 3

Determine the slope of the line.

The points through which the line passes are given as (-2,8) and (4,6). Next, the formula for the slope is given as,

[tex]$m=\frac{y_{2}-y_{1}}{x_{2}-x_{1}}$[/tex]

Substitute [tex]$6 \& 8$[/tex] for [tex]$y_{2}$[/tex] and [tex]$y_{1}$[/tex] respectively, and 4&-2 for [tex]$x_{2}$[/tex] and [tex]$x_{1}$[/tex] respectively in the above formula. Then simplify to get the slope as follows, [tex]$m=\frac{6-8}{4-(-2)}$[/tex]

[tex]$$\begin{aligned}&m=\frac{-2}{6} \\&m=-\frac{1}{3}\end{aligned}$$[/tex]

Step 2 of 3

Write the linear equation in point-slope form.

A linear equation in point slope form is given as,

[tex]$y-y_{1}=m\left(x-x_{1}\right)$[/tex]

Substitute [tex]$-\frac{1}{3}$[/tex] for m,-2 for [tex]$x_{1}$[/tex], and 8 for [tex]$y_{1}$[/tex] in the above equation and simplify using the distributive property as follows, [tex]y-8=-\frac{1}{3}(x-(-2))$\\ $y-8=-\frac{1}{3}(x+2)$\\ $y-8=-\frac{1}{3} x-\frac{2}{3}$[/tex]

Step 3 of 3

Simplify the equation further.

Add 8 on each side of the equation [tex]$y-8=-\frac{1}{3} x-\frac{2}{3}$[/tex], and simplify as follows, [tex]$y-8+8=-\frac{1}{3} x-\frac{2}{3}+8$[/tex]

[tex]$y=-\frac{1}{3} x-\frac{2+24}{3}$$\\ $$y=-\frac{1}{3} x-\frac{26}{3}$$[/tex]

This is the required linear equation.

The endpoints of FG are F(3.75,9) and G (2,-12).
If point E divides FG in the ratio 3:4, the coordinates of point E are what?
If point H divides FE in the ratio 2:3, the coordinates of point H are what?

Answers

Using proportions, it is found that:

The coordinates of point E are (3,0).The coordinates of point H are (3.45, 5.4).

What is a proportion?

A proportion is a fraction of a total amount, and the measures are related using a rule of three.

Point E divides FG in the ratio 3:4, hence:

[tex]E - F = \frac{3}{7}(G - F)[/tex]

This is used to find both the x and y coordinates.

For the x-coordinate, we have that:

[tex]E - F = \frac{3}{7}(G - F)[/tex]

[tex]x - 3.75 = \frac{3}{7}(2 - 3.75)[/tex]

x - 3.75 = -0.75.

x = 3.

For the y-coordinate:

[tex]E - F = \frac{3}{7}(G - F)[/tex]

[tex]y - 9 = \frac{3}{7}(-12 - 9)[/tex]

y - 9 = -9

y = 0.

The coordinates of point E are (3,0).

Point H divides FE in the ratio 2:3, hence:

[tex]H - F = \frac{2}{5}(E - F)[/tex]

This is used to find both the x and y coordinates.

For the x-coordinate, we have that:

[tex]H - F = \frac{2}{5}(E - F)[/tex]

[tex]x - 3.75 = \frac{2}{5}(3 - 3.75)[/tex]

x - 3.75 = -0.3.

x = 3.45.

For the y-coordinate, we have that:

[tex]H - F = \frac{2}{5}(E - F)[/tex]

[tex]y - 9 = \frac{2}{5}(0 - 9)[/tex]

y - 9 = -3.6

y = 5.4.

The coordinates of point H are (3.45,5.4).

More can be learned about proportions at https://brainly.com/question/24372153

#SPJ1

Answer:

The coordinates of point E are (3,0).

The coordinates of point H are (3.45, 5.4).

Step-by-step explanation:

I just got it right on the test

How many sandwiches do
you think would be supplied by this
market if the price were $3.50?

Answers

Answer:  675

=========================================================

Explanation:

Start at $3.50 on the vertical y axis. Draw a horizontal line until you reach the supply curve. Then drop straight down to land somewhere between 650 and 700 units supplied per day. Check out the diagram below.

A good estimate may be to use the midpoint. Add up those values and divide in half.

(650+700)/2 = 1350/2 = 675

Therefore, we estimate that 675 sandwiches per day would be supplied if the price was $3.50 per sandwich.

Simplify 4a - 2a + a

Answers

Answer:

=> 3a

Step-by-step explanation:

=> 4a - 2a + a

=> 2a + a

=> 3a

Answer:

[tex]\boxed{\bf 3a}[/tex]

Step-by-step explanation:

[tex]\bf 4a - 2a + a[/tex]

Combine 4a and -2a = 2a.

[tex]\bf 2a+a[/tex]

Combine 2a and a = 3a

[tex]\bf 3a[/tex]

---------------------------------------

what is 2,104.50 divided by 122

Answers

2104.50 ÷ 122 = 17.25

...

Thirty-three college freshmen were randomly selected for an on-campus survey at their university. The participants' mean GPA was 2.5, and the standard deviation was 0.5. What is the margin of error, assuming a 95% confidence level

Answers

The margin of error is ±0.1706

What is the margin of error?

The margin of error is a statistical solution for the amount of random sampling error in the survey results.

What is 95% of a confidence level?

The 95% confidence interval means from 100 different samples computing a 95% of confidence for each sample. From this, we get the approximate value 95 of the 100 confidence intervals will contain the true mean value.

We have a sample size of n=33

The critical value of the significance level is [tex]\alpha =0.05\\z_{\frac{\alpha }{2} }=1.96[/tex]

Then our sample mean will be 2.5

then the standard deviation is 0.5

from this, we can assume that this is a normal distribution.

then our margin of error will be

[tex]E=[/tex]±[tex]z_{\frac{\alpha }{2} }\frac{deviation}{\sqrt{n} }[/tex]

⇒[tex]E=[/tex]±[tex](1.96)\frac{0.5}{\sqrt{33} }\\[/tex]

≅±0.1706 is the margin of error.

Learn more about the margin of error here:

https://brainly.com/question/14396648

#SPJ4

A sum of Rs. 500 is in the form of denominations of Rs. 10 and Rs. 20. If total number of notes are 35, find the number of notes of each denomination.​

Answers

Answer: 20 of banknotes, denomination of Rs. 10 and 15 of banknotes, denomination of Rs. 20.

Step-by-step explanation:

Let the number of banknotes, denomination of Rs. 10 will be equal to x,

and the number of banknotes, denomination of Rs. 20 will be equal to y.

Hence;   [tex]\left \{ {{x+y=35\ \ \ \ } \atop {10*x+20*y=500\ |:10 \ \ }} \right. \ \ \ \ \left \{ {{x+y=35\ \ (1)} \atop {x+2y=50\ \ (2)}} \right. \\[/tex]

From (2) subtract (1):

[tex]y=15\ \ \ \ \ \Rightarrow\\x+15=35\\x=20.[/tex]

An inequality is shown below:
−np − 3 ≥ 7(c − 4)
Which of the following solves for n?

Answers

The solution of n is [tex]n \le -\frac 1p(7(c - 4) + 3)[/tex]

How to solve for n?

The inequality is given as:

−np − 3 ≥ 7(c − 4)

Add 3 to both sides

−np  ≥ 7(c − 4) + 3

Divide through by -p;

This changes the inequality sign.

So, we have:

[tex]n \le -\frac 1p(7(c - 4) + 3)[/tex]

Hence, the solution of n is [tex]n \le -\frac 1p(7(c - 4) + 3)[/tex]

Read more about inequality at:

https://brainly.com/question/25275758

#SPJ1

I need to find the restrictions for this equation:

Answers

Answer:

Simplify 2x^2-5x-3/x^2-9

The answer is 2x+1 / x+3

Step-by-step explanation:

Simplify the expression.


2. Find four consecutive integers such that six times the sum of the first and third is 12
greater than 9 times the fourth.

Answers

Answer:

9

Step-by-step explanation:

a,a+1,a+2,a+3,...

6(a+(a+2))=12+9(a+3)

6(2a+2)=12+9a+27

12a+12=12+9a+27

12a-9a=27

3a=27

a=9

find the area of a circle whose radius is 'x', unit. ​

Answers

3.14 x² units

Step-by-step explanation:

Radius = x units. ----(given)

π = 3.14

Area of cirlce = πr²

= 3.14 * x²

= 3.14 x² units.

Hope it helps you!!

Answer:

area = πx² square units

Step-by-step explanation:

Formula for the area of a circle:

area = πr²,

where r = radius.

In this problem, the radius is x, so we substitute r in the formula with x.

area = πx² square units

PLEASE HELP ASAP. I WILL MARK AS BRAINLIEST

Answers

You must go 45 miles per hour to reach the longest distance.

Number of hours the car can run

The given parameters are:

Speed = 60 miles per hourTime = 5 hours

When the speed increases by 1 mile per hour, the time reduces by 10 minutes.

This means that:

When

Speed = (60 + x) miles per hour

The time is:

Time = 5 hours - 10 minutes * x

Convert minutes to hour

Time = 5 hours - 1/6 * x hour

This gives

Time = 5 - x/6 hour

Hence, the car can run for (5 - x/6) hours

The distance for 1 tank of gas

We have:

Speed = (60 + x) miles per hour Time = (5 - x/6) hours

The distance is calculated using

Distance = Speed * Time

This gives

Distance = (60 + x)(5 - x/6)

Expand

Distance = 300 - 10x + 5x - x^2/6

Evaluate the like terms

Distance = 300 - 5x - x^2/6

Hence, the distance for 1 tank of gas is 300 - 5x - x^2/6 miles

The longest distance

We have:

D = 300 - 5x - x^2/6

Differentiate

D' = -5 - x/3

Set to 0

-x/3 = 5

Solve for x

x = -15

Substitute x = -15 in D = 300 - 5x - x^2/6

D = 300 - 5(-15) - (-15)^2/6

Evaluate

D = 337.5

Hence, the longest distance is 337.5 miles

The speed for the longest distance

We have:

x = -15

Substitute x = -15 in Time = (5 - x/6) hours

Time = (5 + 15/6) hours

Evaluate

Time = 7.5 hours

The speed is calculated as:

Speed = Distance/Time

This gives

Speed = 337.5/7.5

Evaluate

Speed = 45

Hence, you must go 45 miles per hour to reach the longest distance.

Read more about speed and distance at:

https://brainly.com/question/4931057

#SPJ1

please help me on this question. im not the best at geometry

Answers

Answer: 48

Step-by-step explanation:

By the geometric mean theorem,

[tex]\frac{x}{36}=\frac{64}{x}\\\\x^{2}=36 \cdot 64\\\\x=\sqrt{36 \cdot 64}\\\\x=\sqrt{36} \sqrt{64}=48[/tex]

find the values of x work 10% of my grade

Answers

Answer:

x = 35

Step-by-step explanation:

1. Add all angles

[tex]115+(x+36)+24+(2x+10)+2x[/tex]

= [tex]5x+185[/tex]

2. Match 360 and fix

[tex]5x+185=360[/tex]

[tex]5x=360-185[/tex]

[tex]5x=175[/tex]

[tex]x=175/5[/tex]

[tex]x=35[/tex]

Hope this helps

Answer:

[tex]\boxed{\sf x = 35^o}[/tex]

Step-by-step explanation:

Let's find the value of x :-

[tex]\sf 115 + x + 36 + 24 + 2x + 10 + 2x = 360^o[/tex]

Add the numbers.

[tex]\sf 185 + x + 2x + 2x = 360[/tex]

Combine like terms.

[tex]\sf 5x + 185 = 360[/tex]

Subtract 185 from both sides.

[tex]\sf 5x = 175[/tex]

Divide both sides by 5.

[tex]\sf x = 35^o[/tex]

--------------------------------------

Which expressions are equivalent to 12r-5
Choose all answers that apply:
Choose all answers that apply:

(Choice A)
A
6(2r+(-1))+1

(Choice B)
B
-4(2+(-3r))+3

(Choice C)
C
None of the above

Answers

The expression 12r - 5 are equivalent to 6(2r+(-1))+1 and -4(2+(-3r))+3 option (A) and (B) are correct.

What is an expression?

It is defined as the combination of constants and variables with mathematical operators.

We have an expression:

= 12r - 5

A) 6(2r+(-1))+1

= 12r + 6(-1) + 1

= 12r -5

B) -4(2+(-3r))+3

= -8 -4(-3r) + 3

= -5 + 12r

or

= 12r - 5

Thus, the expression 12r - 5 are equivalent to 6(2r+(-1))+1 and -4(2+(-3r))+3 option (A) and (B) are correct.

Learn more about the expression here:

brainly.com/question/14083225

#SPJ1

The graph below shows a company's profit f(x), in dollars, depending on the price of pens x, in dollars, sold by the company:

Graph of quadratic function f of x having x intercepts at ordered pairs 0, 0 and 6, 0. The vertex is at 3, 120.

Part A: What do the x-intercepts and maximum value of the graph represent? What are the intervals where the function is increasing and decreasing, and what do they represent about the sale and profit? (4 points)

Part B: What is an approximate average rate of change of the graph from x = 3 to x = 5, and what does this rate represent? (3 points)

Part C: Describe the constraints of the domain. (3 points)

Answers

The answers to the various part as well as its reasons are given below

Part A:The x-intercepts shows a zero profit.The maximum value of the graph tells  or depict the maximum profit.The function is one that goes up or increases upward until it reach the vertex and then it falls or decreases after it.This implies that the profit goes up as it reaches the peak at the vertex and it goes down after the vertex up until it gets to zero.The  profits are negative as seen on the left of the first zero and on the right of the second zero.

Part B:

An approximate average rate of change of the graph from x = 3 to x = 5, shows the reduction in profit from  3 to 5.

Part C:

Based on the above,  the domain is one that is held back or constrained by x = 0 .

We are compelled at x = 6 due to the fact that we have to maneuver a negative profit.

Learn more about quadratic function from

https://brainly.com/question/25841119

#SPJ1

While driving, Carl notices that his odometer reads 25,952 miles, which happens to be a palindrome. He thought this was pretty rare, but 2.5 hours later, his odometer reads as 36,563 miles, anoher palindrome. What was Carl's average speed during those 2.5 hours

Answers

Carl's average speed was 4244 miles/hr during those 2.5 hours

First, the odometer showed palindromic no. 25952 miles

after 2.5 hours odometer showed the next palindromic no. is 36563.

hence the distance covered in 2.5 hours = 36563 - 25952 = 110611 miles.

Now, we have to find the average speed with the distance covered 110611 miles in a particular time of 2.5 hours.

Using formula,

average speed = total distance / total time

                    [tex]\frac{10611}{2.5}[/tex] =4244 miles/hour is the average speed

Hence, Carl's average speed was 4244 miles/hr during those 2.5 hours

Learn more about speed and time  here https://brainly.com/question/6504879

#SPJ4

Other Questions
What was your favorite show as a kid? (bubble guppies, mickey mouse, umi zumi, Etc.) Act iii of the tragedy of macbeth serves mainly to a. introduce the plays climax. c. introduce important new characters. b. expose macbeths mounting troubles. d. resolve the plays central conflicts. Sketch the region that is common to the graphs of x 2,y 0, and x + y 6, and find its area. If a population doubles in the course of 100 years, its growth rate would be approximately ________%. In the video, the water level was 15 milliliters before adding the chain, and 20 milliliters after adding the chain. When I weighed the chain, its mass was 67.2 grams.I believe the chain is solid metal (not gold plated), but is probably a blend of gold and other metals. I looked up that gold has a density of 19.3 g/ml, while the other metals like silver and copper usually blended with gold have a combined density of about 9.7 g/ml.Determine what percentage of the chain is gold, by mass, to at least 1 decimal place? Which number sentence matches the question below?For how many days did Curtis practice juggling, if he practiced for 87 hours in all and for 3 hours each day? A. n 87 = 3 B. 3 n = 87 C. 87 n = 3 D. n 3 = 87 Infections caused by a normally nonpathogenic organism often affecting individuals who have a decreased immune system or are elderly are: If a volcano was actively releasing a large amount of volcanic gas and then abruptly stopped, what might happen to the volcano in the near future A member of a group of patrons becomes obviously intoxicated. What should the server do?. If Colorado Springs, Colorado, has 1.2 times as many days of sunshine as Boston, Massachusetts, how many days of sunshine does each city have if there are a total of 482 days of sunshine between the two in a year? Hello there people how is everyone doing.. I was wondering what can you say to someone that means alot to you and you just dont know what to say.. you like the person but its really complicated. So what can you say to make it all right and everything find even if they dont forgive you The express elevator in a skyscraper goes up 51 floors in 123/4 seconds. How many floors does the elevator go up in 1 second The large rectangle below represents one whole.What percent is represented by the shaded area? the sum of all even numbers in 3 digitit will mark as topper After turning the volume all the way up on your speakers, you still cannot hear any sound. Which of the following should be your the next step?OPTIONSYou should replace your system speakers.You should try to re-install your sound card.You should unplug your computer and plug it back in.You should check that your sound isn't muted. Read "The Eagle by Alfred, Lord Tennyson.He clasps the crag with crooked hands;Close to the sun in lonely lands,Ringd with the azure world, he stands.The wrinkled sea beneath him crawls;He watches from his mountain walls,And like a thunderbolt he falls.The rhyming words "hands and "stands emphasizeA. how high up the eagle is.B. how alone the eagle is.C. how old the eagle is.D. how still the eagle is. Question 4 I need help to solve all 3 Genetic modification is used to develop plant and animal species that have new traits. If genetically modified plants are introduced in a natural habitat, they may act in a manner similar to an invasive species. What could be the best solution to prevent them from becoming an invasive species?a.Use pest controlb.Plant more native trees.c.Develop plants that are unable to reproduce.d.Use more fertilizer to promote the growth of genetically modified plants. 2 dogs, 4 horses 1 giraffe and a duck are lying on the bed. 3 chickens are flying over a chair. How many legs are on the ground?. jurgen cuts 5 metre-long timber beam into 3 pieces,first piece 2.34metres second piece 1.85 work out the length of the third piece